0 of 10 Questions completed
Questions:
You have already completed the quiz before. Hence you can not start it again.
Quiz is loading…
You must sign in or sign up to start the quiz.
You must first complete the following:
0 of 10 Questions answered correctly
Your time:
Time has elapsed
You have reached 0 of 0 point(s), (0)
Earned Point(s): 0 of 0, (0)
0 Essay(s) Pending (Possible Point(s): 0)
Average score |
|
Your score |
|
A 46-year-old female presents to the clinic with complaints of recurring headaches. She states that she experiences episodes of sharp pain in her left cheek, which is exacerbated by eating or a cold wind blowing on her face. Outside of these triggers, she is largely pain-free. She doesn’t endorse any associated rhinorrhea, lacrimation, or facial flushing. Which of the following is the most appropriate treatment for her disease?
Which of the following medications is effective in the treatment of myotonia?
An 80-year-old woman with a history of Parkinson’s disease that is well controlled with carbidopa-levodopa presents with her husband for evaluation of delusions. Her husband states that the patient believes that she is being consistently monitored by the FBI and will avoid eating certain foods for fear of being poisoned. She is also constantly worried that her husband will leave her for a younger woman. Which of the following medications would be the most appropriate FDA-approved treatment for these symptoms?
Which of the following is the thalamic relay center for auditory stimuli?
A 32-year-old man with a past medical history of major depressive disorder is brought to the emergency department by his roommate who states that a few days ago the patient stopped taking his medications. Vital signs were within normal limits. On examination, the patient was non-verbal except for occasionally repeating words spoken by the examiner. He would not follow commands. When his arms were positioned off the bed they remained there for several minutes. CT head and CTA of the head and neck were normal. Which of the following is the most appropriate next step in treatment?
A 28-year-old female is brought to the emergency room by her husband because of behavioral changes and memory issues over the last 4 days. Right after the MRI, shown below, the patient had a tonic-clonic seizure. Based on the limited clinical history and imaging provided, which of the following is the most likely diagnosis?
Which of the following is not part of Gerstmann’s syndrome?
Identify the lesion below.
The F wave response is an:
This muscle biopsy is most consistent with which myopathic disease?